The ratio of red M&Ms to blue M&Ms to green M&Ms in a box is 5:3:4. If there are 72 M&Ms in the box, how many of them are blue?

Answers

Answer 1

Answer:

18

Step-by-step explanation:

sum of ratio=5+3+4=12

[tex]blue=\frac{3}{12} \times 72=18[/tex]


Related Questions

What’s the answer pls

Answers

Answer:

168.75 because u put a dot infront of 32 divide it by 54

which expression is equivalent to (2m+3)+(m-1)?
A.5m
B.3m + 2
C.2m+2
D.3m-2

Answers

Answer:

i think the correct is D so 3m-2

Tickets for an upcoming concert are sold out but a local charity is having a raffle and the prize is a pair of tickets to the concert. One hundred people enter the raffle, so each individual who entered has 1/100 probability of winning the pair of tickets. You and a friend both entered. What is the probability that one or the other of you wins the tickets

Answers

Answer:

[tex]P(A\ or\ B) = \frac{1}{50}[/tex]

Step-by-step explanation:

Represent the event that you win with A and the even that your friend win with B.

So, we have:

[tex]P(A) = \frac{1}{100}[/tex]

[tex]P(B) = \frac{1}{100}[/tex]

Required

Calculate P(A or B)

Both events are mutually exclusive.

So:

[tex]P(A\ or\ B) = P(A) + P(B)[/tex]

Substitute values for P(A) and P(B)

[tex]P(A\ or\ B) = \frac{1}{100} + \frac{1}{100}[/tex]

Take LCM

[tex]P(A\ or\ B) = \frac{1+1}{100}[/tex]

[tex]P(A\ or\ B) = \frac{2}{100}[/tex]

Simplify:

[tex]P(A\ or\ B) = \frac{1}{50}[/tex]

triangle sum theorem
i will give brainliest

Answers

Answer:

The answer is b=128

Step-by-step explanation:

25+27=52

180-52=128

Answer:

∠A + ∠B + ∠C = 180°

⇒ 25° + 27° + ∠B= 180°

⇒ 52° + ∠B= 180°

Subtract both sides by 172°

⇒ 52° – 52° + ∠B = 180° – 52°

Therefore, ∠B = 128°

a sequence of payment made at equal time period is a/an'????​

Answers

Answer:

Annuity.

Step-by-step explanation:

A sequence of payment made at equal time period is called an annuity.

Basically, annuity can be calculated using the compound interest formula. It is given by the mathematical expression;

[tex] A = P(1 + \frac{r}{n})^{nt}[/tex]

Where;

A is the future value.

P is the principal or starting amount.

r is annual interest rate.

n is the number of times the interest is compounded in a year.

t is the number of years for the compound interest.

Additionally, the time period between each payment is called payment period.

The term of an annuity refers to the time from the beginning of the first payment made by an individual to the end of the last payment period.

45x - 18 factored i really need help

Answers

45x - 18 factored I really need help
Can be rewritten as
9(5x--2)

527856 \ 8

How do I solve this with long division to find my quotient?

Answers

Answer:

65,982

Step-by-step explanation:

Look at the attached image for the step by step solution

hich expression is equivalent to -4(7 - 2m) + 11m?

Answers

Answer:

[tex] - 4(7 - 2m) + 11m \\ = - 28 + 8m + 11m \\ = \boxed{19m - 28}[/tex]

Expression (19m-28) is equivalent to -4(7 - 2m) + 11m.

Can anyone help me pls

Answers

75%

Explain: count all the items in the list then count the caps and yo-yos together.
That’ll be 6/8 which is 75%
75% because there is 6/8 you will get the yo yo or the other one. so you put 6/8 = ?/100. you multiply 100 with 6 then divide 600 with 8

PLEASE HELP ASAP!!! awarding a lot of points!

Answers

Exact Form:

x=5/4

Decimal Form:

x=1.25

Mixed Number Form:

x= 1 1/4

The correct answer is,

[tex]x = \frac{5}{2} [/tex]

Can someone help me?

Answers

Answer:

with what

Step-by-step explanation:

dont see nothinfg

Answer:BD equals CE? Are lines parallel

Step-by-step explanation:

Paul is a newspaper boy who received a total piecework
paycheck of $265.72. He receives 73 cents for every
newspaper he delivers. How many newspapers did he
deliver?

Answers

Answer:

364

Step-by-step explanation:

265.72 ÷ 0.73 is 364

Check:

364 x 0.73

265.72

Bea is buying pens and pencils. Pens cost $2 and pencils cost $1. She plans on spending no more than $10 and wants to buy at least 10 items. Which two inequalities model this situation?

Answers

Well each pen costs $2 and each pencil costs $1.

One of the equations would be

2p + 1c ≤ 10

She doesn’t want to spend more than 10 dollars but wants to get 10 items. So she’ll be spending $10. For the pencils.

1c ≤ 10 c = 10

1(10) ≤ 10

10 ≤ 10

Now for the pens

2p ≤ 10

2. 2

P = 5

5 ≤ 10

Anymore will go over her budget. She wants to buy at least 10 items not 10 exactly so she can buy 5 pens or 10 pencils.

The first floor of a tiny house has the length of 11 feet. The width of the kitchen feet is 7 feet and the width of the bottom bathroom feet is 4 feet the expression 11(7+4) represents the total area in the square feet.

why an expression to represent a total area as a some of the areas of each room

Answers

the blanks should be 7 and 4

Answer:

11 and 4

Step-by-step explanation:

9/12 = 3/D O D = 6 O D = 4 O D = 5 O D - 10

Answers

Answer:

The answer would be D = 4 or option 2.

Step-by-step explanation:

9/12 = 3/?

First, you want to simplify 9/12

9/12 = 3/4

This shows you that D = 4

Coupd someone help me with this? (picture)​

Answers

Answer:

2x3 = 6

6x14 = ...

Step-by-step explanation:

All the solutions with a + in them are not applicable to this multiplication

if a box of oranges weigh 4 kilograms one orange weighs 80g how many oranges?

Answers

There are 50 oranges in the box.

Solve the problem using long division

Answers

Answer and Step-by-step explanation:

[tex](m^{2} - 3m - 7) (\frac{1}{m+2})[/tex]

       

        1m     -5

         ____________

[tex]m+2 |m^{2} - 3m - 7[/tex]

    - m^2 + 2m

___________________

               -5m   - 7

            - -5m   - 10

___________________

                           3

(The negatives cancel out).

[tex]m - 5 + \frac{3}{m+2}[/tex]   <- This is the answer.

#teamtrees #WAP (Water And Plant)

the jet stream is a wind current that flows across the US from west to east. Flying with the jet stream, an airplane flew 2700 miles in 4.5 hours. Against the same wind, the return trip took 6 hours. Find the speed of the plane in stil air and the speed of the jet stream.

Answers

Answer:

The speed of the plane in still air and the speed of the jet stream are 525 miles per hour and 75 miles per hour.

Step-by-step explanation:

From Mechanical Physics, we know that absolute velocity of the airplane ([tex]v_{A}[/tex]) is equal to sum of the absolute velocity of the jet steam [tex](v_{J})[/tex] and the relative velocity of the airplane with respect to jet stream (velocity of the airplane in still air) ([tex]v_{A/J}[/tex]). Al velocities are measured in miles per hour. Let suppose that both jet stream and airplanes travels at constant velocity. Now, we construct the following system of linear equations:

Flight with the jet stream

[tex]v_{J}+v_{A/J} = \frac{2700\,mi}{4.5\,h}[/tex]

[tex]v_{J}+v_{A/J} = 600\,\frac{mi}{h}[/tex] (1)

Flight against the jet stream

[tex]v_{J} -v_{A/J} = -\frac{2700\,mi}{6\,h}[/tex]

[tex]v_{J}-v_{A/J} = -450\,\frac{mi}{h}[/tex] (2)

The solution of this system of linear equations is:

[tex]v_{J} = 75\,\frac{mi}{h}[/tex], [tex]v_{A/J} = 525\,\frac{mi}{h}[/tex]

The speed of the plane in still air and the speed of the jet stream are 525 miles per hour and 75 miles per hour.

PLS HELP!!!! WILL GIVE BRAINLIEST!!!!

Write the slope-intercept inequality for the graph below. If necessary, use <=
fors or >= for
(0,4)
(-3,0)

Answers

(3,0) (-4,0)

not for sure but still mark brainliest cause i need it ✋ im doing bad rn

Simplify y ^2 over 6y^5.

Answers

Answer:

1/6y^3

Step-by-step explanation:

Answer:

1/6y^3

Step-by-step explanation:

cancel the common factor of y^2 and y^5

40 POINTS PLEASE HELP

Answers

Answer:

Step-by-step explanation:

Y = 10

C = 40

D = 76

Answer: Okay found the answer hope I am not too late

Step-by-step explanation:

Y is 10

C is 40

D is 76

Hope this helps!

The sum of three consecutive even integers is 486. find all three integers​

Answers

160 and 162 and 164

Step-by-step explanation:

To find the three even integers, use the equation

x + (x + 2) + (x + 4) = 486

Then solve:

Add the common terms (the xs and numbers)

3x + 6 = 486

Subtract the 6 from both sides

3x = 480

Then divide

x = 160

So 160 + (160 + 2) + (160 + 4) = 486

160 + 162 + 164 = 486

So your three consecutive even integers are

Can someone help pls ASAP running out of time

Answers

Answer:  The correct answer is A

Step-by-step explanation:

If your multiplying fractions you take the first fraction 1/4 and the number 5, but you turn that into a fraction so now it's 5/1, and you multiply them together to get 5/4

1/4 x 5/1 = 5/4

✩brainliest appreciated ✩

Given the following formula, solve for v.
S=1/2a2v+c

Answers

Answer:

Since there is no initial condition, the exact value of v cannot be determined, but you can set the equation up for a general evaluation of the situation.

s = 1/2a^2*v+c

First you need to subtract c from both sides to get

s-c = 1/2a^2*v

then you can just divide both sides by 1/2a^2 to get v

(2(s-c))/a^2=v

when dividing a fraction, such as 1/2, make sure to keep in mind that you're really multiplying the flipped version, so that dividing by 1/2 means multiplying by 2.

Step-by-step explanation:

+
2
8
2
Least multiple that is the sam
Add using renamed fractions

Answers

Answer:

no

Step-by-step explanation:

Could someone answer and explain. Will mark the brainliest

Answers

Answer: I think you’re right??

Step-by-step explanation:

Factor the trinomial. Factor out the GCF first.
11x^2 + 33x + 22

Answers

Step-by-step explanation:

11x² + 33x + 22

= 11(x² + 3x + 2)

= 11(x + 1)(x + 2).

List all the factors.
48

Answers

Answer:

Postive intiger factors: 1, 2, 3, 4, 6, 8, 12, 16, 24 and 48.

Answer:

1,2,3,4,6,8,12,16,24,48

Step-by-step explanation:

Mia went to the art store and bought 2 poster boards and 3 markers. She spent A total of $6.50. The total tax was $1.25. How much did each marker cost? *

Answers

The answer is $1.05 but this is only right if the posters cost the same amount as the markers.
Other Questions
On the first day of January, Harris Company borrowed $3,000 on a one-year note payable bearing interest at 5% per year. The note specifies that principal and interest must be paid in full at the end of the one-year period. On June 30, the adjusted trial balance will show Interest Payable of please help due asap jdbhdbdbd A change of state is a(n)process.A. irreversible B. reversible Malachy was thinking of a number. Malachy halves the number and gets an answer of 67.3.Form an equation with x from the information. Charlin got 20 questions correct on her exam and scored a 80%. How many (total) questions were on the exam 4.1% of 30Im in 7th grade so please dont answer it to complicated lol Was it possible for the Contederacy win a war of attrition against the Union, why or why not ? what are three aspects of french culture that are similiar to the united states? What happened as a result of the Creek War?Many Creeks gave up their lands and relocated to Florida.The Creeks successfully defeated Jackson and remained in Alabama.Most Creeks relocated to Indian Territory.The Creeks signed a treaty that allowed them to share land ownership with white settlers. PLS HELP !!!About how far away is Korea from Japan by water?A. 20 miles B. 300 miles C. 100 miles Im failing please help! A company that makes shopping carts for supermarkets and other stores recently purchased some new equipment that reduces the labor content of the jobs needed to produce the shopping carts. Prior to buying the new equipment, the company used 6 workers, who together produced an average of 100 carts per hour. Workers receive $11 per hour, and machine cost was $40 per hour. With the new equipment, it was possible to transfer one of the workers to another department, and equipment cost increased by $12 per hour, while output increased by 4 carts per hour. a. Compute labor productivity under each system. Use carts per worker per hour as the measure of labor productivity. (Round your answers to 3 decimal places.) mi casa esta cerca_ escuela write the equation of the line that is parallel to y = 4x -7 and passes through the point (-2, -5). show your work. how did people think Washington should be the country's first president What is 20% of 800, if 10% of 800 is 80? Please help with this. How many factors does a scientist want to differ between the experimental and control groups?A) 2B) 0C) 1D) 3 The equations X+5y = 10, 3x-y = 1, X-5y = 10, and 3x+y = 1 are shown on the graph below. Which is the approximate solution for the system of equation x+5y=10 and 3x+y=1?O (-.3, 2.1)O (-0.3, -2.1)O (0.9, -2.1)O(0.9, 1.8) 5. Pueden ustedes or el telfono?-S,podemos or.6. Puedes ver el mar?-No, nopuedo ver.